LSAT and Law School Admissions Forum

Get expert LSAT preparation and law school admissions advice from PowerScore Test Preparation.

 Administrator
PowerScore Staff
  • PowerScore Staff
  • Posts: 8916
  • Joined: Feb 02, 2011
|
#64913
Complete Question Explanation

Must be true. The correct answer choice is (B)

The two sentences produce the following conditional relationships, which can be linked:


..... ..... AS = angle stable
..... ..... GI = gravitational influence of Earth’s large, nearby Moon
..... ..... PCE = planet’s climate too extreme and unstable to support life

..... ..... Sentence 2: ..... AS :arrow: GI
..... ..... Sentence 3: ..... PCE :arrow: AS
..... ..... Chain: ..... ..... PCE :arrow: AS :arrow: GI


Again, consider the answers that are most likely to appear in a problem like this: either the chain inference PCE :arrow: GI, or the contrapositive of that chain inference, GI :arrow: PCE.

Answer choice (A): This incorrect answer is the Mistaken Reversal of the chain inference. The diagram for this answer choice would be as follows, with the sub-M indicating Mars:


..... ..... GIM :arrow: PCEM


Answer choice (B): This is the correct answer. As expected, this is the contrapositive of the chain inference.

Answer choice (C): This incorrect answer is the Mistaken Reversal of the third sentence, and is diagrammed as follows:


..... ..... AS :arrow: PCE


Answer choice (D): This answer discusses issues that were not raised in the stimulus, and is thus incorrect.

Answer choice (E): The stimulus indicates that Earth’s large Moon has been necessary for the stable angle of Earth’s tilt, and this stable angle has been necessary for a climate that can support life. Mars, with many small moons, tilts at fluctuating angles and cannot support life. The key difference is that Earth’s Moon is large, and that creates a greater gravitational influence. It is possible, therefore, that a planet with more than one moon could have a stable angle as long as at least one of the moons was of sufficient size (in Mars’ case, the stimulus indicates each moon is small). Thus, it is possible that a planet can have more than one moon and support life. This scenario is contrary to the answer choice, and thus this answer is incorrect.

Note, this question appears in the Logical Reasoning Bible, and thus it has also been discussed multiple times on this Forum. For further discussions, please visit:

lsat/viewtopic.php?t=27721

lsat/viewtopic.php?t=10839

lsat/viewtopic.php?t=1868

lsat/viewtopic.php?t=28794
 srcline@noctrl.edu
  • Posts: 243
  • Joined: Oct 16, 2015
|
#23698
Hello,

So I am having a hard time linking all these conditional statements. Here is my diagram:

1. < kept fairly stable :arrow: grav. inf. Earth's nearby moon
(+) If no grav. inf. by Earths nearby moon :arrow: < will not be kept fairly stable

2. w/o stable and mod. axis tilt :arrow: planet unable to support life

(+) if planet able to support life :arrow: w/ stable or moderate axis tilt

Conclusion: Mars has small moons, tilts at widely fluctuating angles and cant support life. I am not seeing how I can link these conditional statements up. Maybe there's so much going on here I'm getting confused. I picked answer choice A.

Thankyou
Sarah
 David Boyle
PowerScore Staff
  • PowerScore Staff
  • Posts: 836
  • Joined: Jun 07, 2013
|
#23787
srcline@noctrl.edu wrote:Hello,

So I am having a hard time linking all these conditional statements. Here is my diagram:

1. < kept fairly stable :arrow: grav. inf. Earth's nearby moon
(+) If no grav. inf. by Earths nearby moon :arrow: < will not be kept fairly stable

2. w/o stable and mod. axis tilt :arrow: planet unable to support life

(+) if planet able to support life :arrow: w/ stable or moderate axis tilt

Conclusion: Mars has small moons, tilts at widely fluctuating angles and cant support life. I am not seeing how I can link these conditional statements up. Maybe there's so much going on here I'm getting confused. I picked answer choice A.

Thankyou
Sarah
Hello Sarah,

Mars is an example, not a conclusion, as noted by the words "Mars, for example".
You can link the conditionals by saying something like

planet support life (subscript Earth) :arrow: stable/moderate tilt :arrow: Moon gravity.

The contrapositive, if the Moon is missing, shows Earth can't support life, so answer B.

Hope this helps,
David
 aatakyi1
  • Posts: 4
  • Joined: Mar 23, 2017
|
#34390
hello,

I'm still having trouble trying to figure out the contrapositive to this question. It is very confusing to me. Can you kindly walk me thru step by step? Thank you!

Adwoa
User avatar
 Jonathan Evans
PowerScore Staff
  • PowerScore Staff
  • Posts: 726
  • Joined: Jun 09, 2016
|
#34394
Aatakyi,

I'm right there with you! This is a difficult question to follow. Let's parse it out. Focus on the pertinent statements, those which convey some kind of logical relationship.
  1. Stable angle ONLY IF gravitational influence (SA :arrow: GI), contrapositive GI :arrow: SA
  2. If no stable angle, then climate too extreme (SA :arrow: EC), contrapositive EC :arrow: SA
The rest of the stimulus is kinda "filler." It might be useful, but it's not the crux of the argument.

Let's see how we can connect these two conditional statements together. We can accomplish this by connecting either statement to the other's contrapositive:
  • GI :arrow: SA :arrow: EC

    OR

    EC :arrow: SA :arrow: GI
In other words:
  • If no gravitational influence, then no stable angle, then extreme climate.

    OR

    If not extreme climate, then stable angle, then gravitational influence.
Answer Choice B corresponds to the first relationship outlined above. I hope this helps!
 aatakyi1
  • Posts: 4
  • Joined: Mar 23, 2017
|
#34476
yes thank you
 Mi Kal
  • Posts: 48
  • Joined: Jun 10, 2017
|
#36839
Hi.

I see the link between the Sufficient and Necessary Conditions. But, for some reason, I cannot get to answer B. How does the general (lack of gravitational influence and stability coupled with the extreme climate) translate to the specific (Moon leaving Earth's orbit)? In the stimulus, nothing is even mentioned of that being a possibility.

Thanks.

Michael
 Adam Tyson
PowerScore Staff
  • PowerScore Staff
  • Posts: 5153
  • Joined: Apr 14, 2011
|
#36896
Hi Mi Kal, let me try to help you out a bit here. It's not about general to specific, because the stimulus is already specific. The only reason we can maintain a stable tilt is because we have our large moon. Without that moon, the tilt would not be stable. That's entirely specific to our Earth-Moon system.

The general rule is that any planet that does not have a stable, moderate tilt, like Earth's, cannot support life. That general rule applies to all planets, including Earth. If we didn't maintain our stable, moderate tilt, we couldn't support life.

So, put that all together - if the moon were to leave, we could no longer maintain that stable, moderate tilt. Without that tilt, we couldn't support life. Hence, without the moon, no life on Earth.

Just to be clear, we are not trying to prove that the moon will leave, or even that it could. We just want to show that if it did somehow leave us, we'd be dead.
 Mi Kal
  • Posts: 48
  • Joined: Jun 10, 2017
|
#36898
Thanks Adam.
 olafimihan.k
  • Posts: 25
  • Joined: Jul 04, 2017
|
#39429
Hello,

Can you explain the difference between answer choices A and B?

Thanks

Get the most out of your LSAT Prep Plus subscription.

Analyze and track your performance with our Testing and Analytics Package.